Explanation
Can someone explain the right answer?
zia305 on December 20 at 04:04AM
  • October 2004 LSAT
  • SEC3
  • Q17
1
Reply
Why can't the third site be visited by F?
Couldn't the order be O, F, F, F, G (w/ the centuries being 8, 9, 9, 9, 10 respectively)? the sec...
Vennela-Vellanki on December 20 at 03:56AM
  • October 2004 LSAT
  • SEC3
  • Q14
1
Reply
Last Rule
For the last rule, why does "either 1st or 4th" mean that the third spot is more recent than both...
gracehazelhoff on October 21 at 02:59PM
  • October 2004 LSAT
  • SEC3
  • Q16
1
Reply
last rule
The last rule says, "The site visited third dates from a more recent century than does either the...
AndrewArabie on February 22, 2023
  • October 2004 LSAT
  • SEC3
  • Q14
2
Replies
Answer choice
Why can't F be third? I got O-F-F-G-O which is respectively; 8-9-9-10-8. Where is this wrong? ...
Melody on January 30, 2021
  • October 2004 LSAT
  • SEC3
  • Q14
2
Replies
Sad!
Pretty sad to have shelled out 1200 bucks for this study system and I had to google how to do thi...
jamesteemartin on May 13, 2020
  • October 2004 LSAT
  • SEC3
  • Q13
2
Replies
Question about setup...
What is the fastest way to determine if a game is worth framing scenarios or just going right int...
DrKumar on December 4, 2019
  • October 2004 LSAT
  • SEC3
  • Q1
3
Replies
Why not B?
Hi I was between B and E but both seemed must be true in this case for me. Could you please ex...
farnoushsalimian on October 15, 2019
  • October 2004 LSAT
  • SEC3
  • Q4
1
Reply
Could you,show me how to diagram this game?
Thanks! You guys are great.
Jprince on November 7, 2014
  • October 2004 LSAT
  • SEC3
  • Q20
1
Reply